Questions tagged [ca.classical-analysis-and-odes]

Special functions, orthogonal polynomials, harmonic analysis, ordinary differential equations (ODE's), differential relations, calculus of variations, approximations, expansions, asymptotics.

Filter by
Sorted by
Tagged with
1 vote
0 answers
150 views

A generalization of polynomials in one variable

Let us consider the space of polynomials $P^N$ of degree $\le N$. If $f\in P^N$ vanishes in $>N$ points, then $f\equiv 0$, but for any $N$ points, or fewer, there exists $f\neq 0$ vanishing at ...
user90189's user avatar
  • 398
3 votes
1 answer
630 views

What is the Fourier series of $\sin(1/x)$ in $[-\pi,\pi]$?

What is the Fourier series of $\sin(1/x)$ (or $x^k\sin(1/x)$, where $k$ is a positive integer) in $[-\pi,\pi]$? This function evidently does not satisfy Dirichlet's conditions. However, Dirichlet's ...
user37022's user avatar
1 vote
1 answer
128 views

Time varying domain in Chen Wenxiong and Li Congming 's study on $-\Delta u=\exp u$ in $\mathbb R^2$ and $\int_{\mathbb R^2}\exp u(x) \, dx< +\infty$

I'm considering a problem about time varying domain in Chen Wenxiong and Li Congming 's study on $-\Delta u=\exp u$ in $\mathbb R^2$ and $\int_{\mathbb R^2} \exp u(x) \, d x< +\infty$. LEMMA 1.1 (...
Elio Li's user avatar
  • 729
6 votes
1 answer
735 views

Twisted Riemann sums

Let $f(x)$ be a real-valued Riemann integrable function supported in $[0,1]$ with range in $[0,1]$. Let $\alpha$ be irrational. Consider the weighted Riemann sum $$S_N:=\frac{1}{N}\sum_{k=1}^Nf\left(\...
user499631's user avatar
3 votes
0 answers
103 views

Regularity of solutions to an integral ODE

I'm trying to figure out the regularity for solutions to the following integral equation: \begin{align} \begin{cases} \displaystyle{\frac{d}{dt}}u(t,x)&=\displaystyle{\frac{1}{\sigma_d\epsilon^{d+...
hamath's user avatar
  • 121
1 vote
0 answers
61 views

$L^p$ norm of Fourier transform of function composed with a diffeomorphism

Suppose $f$ is a compactly supported smooth function from $\mathbb{R}^n$ to $\mathbb{C}$ and $A$ is a diffeomorphism on $\mathbb{R}^n$, do we have any theorems relating the $L^p$ norm of $\hat{f}$ and ...
Simplyorange's user avatar
8 votes
3 answers
1k views

Are all positive eigenfunctions principal eigenfunctions?

In a given domain $\Omega$, we have: $\Delta u=-\lambda u$ with $u>0$. Does this mean that $u$ is a principal eigenfunction for $\Delta$ in $\Omega$? Also, more generally, does this also apply for $...
user734979's user avatar
3 votes
1 answer
134 views

Recover an $L^1$ integrand by partial differentiation

Denote by $m$ the 2-dimensional Lebesgue measure on $\mathbb{R}^2$. Let $f$ be an element of $L^1(m)$ that takes only nonnegative values. Define $F : \mathbb{R}^2 \rightarrow [0,\infty)$ by $$F(x,y) = ...
w116c576's user avatar
2 votes
1 answer
196 views

Asymptotics for oscillatory integral

Consider the following integral for $f \in C_c^{\infty}(\mathbb R^n)$, $x_0$ fixed (possibly zero), and $n \ge 3$ $$F(\lambda) = \int_{\mathbb R^n} e^{i\lambda \vert x-x_0 \vert^2} \frac{f(x)}{\vert x ...
António Borges Santos's user avatar
3 votes
0 answers
150 views

Number of positive roots for an exponential sum

Given $n\geq 3$ distinct constants $c_1, c_2, ..., c_n \in\mathbb{C}$, I want to bound/estimate the number of positive real roots for the equation $$f(x):=\sum_{i=1}^{n}\dfrac{c_i^n}{\prod_{j\neq i}(...
Abhishek Halder's user avatar
-4 votes
1 answer
106 views

Uncountable Cantor's diagonal argument on $S^2$ [closed]

Let $F: S^2 \rightarrow \mathbb{R}^2$ be a continuous function. Does there exist a unit vector $v \in \mathbb{R}^2$ and a continuous function $f(x):S^2\rightarrow \mathbb{R}$ such that $f(x)>0$ on $...
MathLearner's user avatar
1 vote
0 answers
66 views

Estimating commutator of Fourier integral

Let $f(x)= \log(\vert x\vert)$ on $\mathbb R^2$ and define $s_n:H^2 \to L^2$ where $H^2$ is the second Sobolev space by $$ s_n(g)(x) = \frac{nf(x)}{4\pi i} \int_{\mathbb R^2} e^{\frac{in\vert x-y\...
António Borges Santos's user avatar
2 votes
0 answers
73 views

Verify the explicit solution formula for a degenerate Fokker-Planck equation $\partial_t u = \nabla\cdot(D\,\nabla u + Cxu)$

Consider the following degenerate Fokker-Planck equation in $\mathbb{R}^d$ $$\partial_t u = \nabla\cdot(D\,\nabla u + Cxu),\quad u(t=0) = u_0 \label{1}\tag{1}$$ where $D \in \mathbb{R}^{d \times d}$ ...
Fei Cao's user avatar
  • 700
4 votes
1 answer
155 views

Uniqueness of critical points for Lipschitz perturbations of uniformly convex Hamiltonians

Consider a macroscopic free energy functional of the form $$\mathcal{F}_\beta(\mu):= \frac{1}{\beta}\int_{\mathbb{R}^d}\log(\mu)\mu dx + \int_{\mathbb{R}^d}V(x)\mu(x)dx + \iint_{(\mathbb{R}^d)^2}g(x-y)...
Matt Rosenzweig's user avatar
1 vote
0 answers
94 views

Regularity of solution to heat equation

If $u$ is solution of $u_{t}=\Delta u$ in a bounded domain $\Omega$, is it true that: $\sum_{\lvert \alpha\rvert=2k}\|D^{\alpha}u\|_{L^{2}(\Omega)}^{2}\leq |\Delta^{k}(u^{(l))}\|_{L^{2}(\Omega)}^{2}=\...
Amira's user avatar
  • 11
3 votes
1 answer
215 views

Reference request: analysis of a nonlinear Fokker-Planck type equation

It is well-known that the linear Fokker-Planck equation (written in one space dimension for simplicity) $$\partial_t \rho = \partial_x \left(\rho_\infty \partial_x\left(\frac{\rho}{\rho_\infty}\right)\...
Fei Cao's user avatar
  • 700
0 votes
0 answers
36 views

Conditions for an ODE with convolution term to have a probability distribution solution

Suppose we have a simple ODE like: $$ y''(x) + 2ay'(x) + by(x) = 0 $$ with the condition $y(0)=0$ for $x\leq 0$. Then the solution on $(0,\infty)$ will be of the form $Axe^{-ax}$ when $a^2=b$, $Ae^{-...
user1598's user avatar
  • 177
0 votes
0 answers
47 views

When is the derivative of an ODE solution wrt the initial condition a simple exponential?

Suppose we are given an autonomous ODE $y' = f(y)$ such that $y$ is smooth with respect to the initial condition. We then have $$(\nabla y)' = \nabla f(y) \nabla y.$$ If $\nabla f(y(t)) $ commutes ...
Stefan Perko's user avatar
0 votes
1 answer
83 views

On the validity of a certain Grönwall-type inequality

Assume that $u~ \colon \mathbb{R}_+ \to [-M,M]$ is a bounded continuously differentiable function such that $u(0) = 0$ and $$u(t) \leq \int_0^t \lambda(s)~u(s)~\mathrm{d}s + C \label{1}\tag{1}$$ where ...
Fei Cao's user avatar
  • 700
2 votes
1 answer
351 views

What are the best definitions for smoothness of a 2D curve (real-valued function)?

Sounds like a trivial question, but could not find any answer other than the fact that there are many ways to define it. My problem is this: I look at different elevation maps, some with sharp ...
Vincent Granville's user avatar
0 votes
1 answer
136 views

How are the Legendre Polynomials of second kind for negative degrees defined?

For a script I have to evaluate the associated Legendre polynomial of second kind $Q^0_{n}(z)$. Until now, I was using an implementation that is based on the definition in equation 8.702 in the Book &...
BenjaminBluemchen's user avatar
1 vote
0 answers
20 views

Asymptotic behaviour of the solution to some delayed ODE

Following the previous post Asymptotic behaviour of the solution to some delayed stochastic differential equation I consider a deterministic version (as no answer is received) : $$\frac{d x^\theta}{d ...
Fawen90's user avatar
  • 975
9 votes
2 answers
644 views

Asymptotic behavior of a certain oscillatory integral

Let $x>0$ and consider the integral $$I(x):=\int_0^\infty \frac{e^{i r}}{r^{\frac{1}{2}}} \int_0^\infty \frac{e^{-s}}{s^{\frac{1}{2}}} \frac{r}{sx+\sqrt{sxr}+r} \, ds \, dr.$$ I am trying to ...
Medo's user avatar
  • 744
7 votes
2 answers
644 views

For a manual evaluation of a definite integral

I note that Mathematica could yield the identity $$\int_0^1\frac{\log(1+x^2(x-1)/2)}{x^2(x-1)}dx=\frac{\pi(\pi-4)-12\log^22+24\log2}{16}.\tag{1}\label{1}$$ But I don't know how Mathematica got this. ...
Zhi-Wei Sun's user avatar
  • 14.5k
3 votes
1 answer
194 views

An ODE for tensor - possibility of the equation together with the initial condition at $t=0$ deciding the solution for all $t>0$

Let $A_{ijl}(t,x) : [0,\infty) \times \mathbb{R}^n \to [\mathbb{R}^n]^3$ be a smooth tensor field. That is, $i,j,l \in \{1,2,3, \cdots, n\}$ Further assume that $A_{ijl}(t,x)=A_{jil}(t,x)$ for all $(t,...
Isaac's user avatar
  • 2,749
1 vote
1 answer
63 views

Upper bounds for the spatial differential of the inverse of a flux

It is well known that given a regular velocity field $b: \mathbb{R} \times \mathbb{R}^n \to \mathbb{R}^n$ (say, continuous in time and uniformly Lipshitz in space), the flux $X$ associated to $b$ is a ...
tommy1996q's user avatar
1 vote
0 answers
108 views

Relating $f(x)$ to its Laplace Transform for values other than $x=0$?

Suppose $X\in (0,1]$ is a random variable where $f(x)$ is its CDF and $g(t)$ is the Laplace Transform of $f(x)$. Tauberian theorems (Theorem 2.3 in Coqueret's "Approximation of probabilistic ...
Yaroslav Bulatov's user avatar
11 votes
2 answers
831 views

What are the iterates of $x \mapsto 1 - \sqrt{1-x^2}$?

Let $f\colon [0,1]\to[0,1]$ be given by $f(x) = 1-\sqrt{1-x^2}$, i.e., the increasing auto-homeomorphism of $[0,1]$ whose graph is a quarter circle centered at $(0,1)$. I am interested in what can be ...
Gro-Tsen's user avatar
  • 30.2k
1 vote
0 answers
88 views

Notation for right hand side of local smoothing conjecture

In Tao's "Recent progress on the restriction conjecture" On page 53, Tao introduced the local smoothing conjecture: let $u(t,x)$ be the solution to the wave equation $u_{tt}=\Delta u$, $u(0,...
Simplyorange's user avatar
4 votes
1 answer
220 views

A lower bound for the $L^1$ norm of real trigonometric polynomials

This question is somewhat similar to Minimizing the L1 norm of odd-term trigonometric polynomial. The context of the question is based on the paper Hardy's Inequality and the $L^1$ norm of Exponential ...
johng23's user avatar
  • 270
0 votes
0 answers
46 views

What fractional equation does this convolution solve

Assume that $f:\mathbb{R}\to \mathbb{R}$ is an infinitely differentiable with compact support and let $E_{\alpha}$ be the one-parameter Mittag-Leffler function with $0<\alpha<1$. Find the ...
Medo's user avatar
  • 744
0 votes
0 answers
96 views

Asking a reference about the $p$-Laplacian of $|\nabla u|^p$

It is well-known that for a harmonic function $u$, i.e. $$ \Delta u=0, $$ the quantity $|\nabla u|^2$ is subharmonic, i.e. $$\Delta (|\nabla u|^2) \geq 0. $$ Reason: $$\Delta (|\nabla u|^2)= 2 \nabla (...
Hheepp's user avatar
  • 361
0 votes
1 answer
194 views

Series involving sine and cosine

Let $(a_n)_n$ be an increasing real sequence with $a_n=O(\sqrt n)$. Is it true that there exists an increasing function $\phi:\mathbb N\to\mathbb N$ such that $$\lim \left|\sum\limits_{k=1}^{\phi(n)}\...
Dattier's user avatar
  • 3,801
1 vote
1 answer
210 views

Approximation for a Bessel function integral

I'm trying to calculate hit probabilities on a dart board if the dart thrower has some Gaussian angle distribution function with width $\Delta$ and some systematic angle offsets $\phi_0, \theta_0$. ...
mzw's user avatar
  • 19
4 votes
0 answers
125 views

What can be possible conditions for the solution of an autonomous ODE to be conservative with respect to the initial data?

Let $F : \mathbb{R}^n \to \mathbb{R}^n$ be a smooth mapping and consider the following autonomous ODE \begin{equation} y'(t)=F(y(t)) \end{equation} with the initial data $y(0)=x \in \mathbb{R}^n$. ...
Isaac's user avatar
  • 2,749
5 votes
1 answer
308 views

Long tail property of Laplace transforms

A function $F: \mathbb R_+ \rightarrow \mathbb R_+$ is said to be long tailed if $F(\infty)=0$ and for all $y \geq 0$ $$\frac{F(x+y)}{F(x)} \rightarrow 1, \quad x\rightarrow \infty.$$ Let $\mu$ be a ...
Mr_3_7's user avatar
  • 105
2 votes
0 answers
68 views

Extremizing the integral part of an integro-differential equation

Consider the problem of finding a continuously twice-differentiable function $x(t)$ which extremizes the convergent improper integral \begin{equation} I=\int_{-\infty}^{t} f(x,s)\mathop{ds} \end{...
UNOwen's user avatar
  • 79
4 votes
0 answers
89 views

Are Sobolev isometries in Minkowski space smooth

Let $\Omega\subset\mathbb{R}^d$ be an open regular domain and let $f\in W^{1,\infty}(\Omega;\mathbb{R}^d)$ satisfy that $df\in\operatorname{SO}(d)$ almost-everywhere. It was proved by Reshetnyak (in a ...
Raz Kupferman's user avatar
12 votes
0 answers
271 views

Can a 4D spacecraft, with just a single rigid thruster, achieve any rotational velocity?

(Copied from MSE. Offering four bounties over time, I got no response, other than twenty-nine upvotes.) It seems preposterous at first glance. I just want to be sure. Even in 3D the behaviour of ...
mr_e_man's user avatar
  • 191
2 votes
1 answer
101 views

If $b\in C^1(E, \mathbb{R})$ and $b'$ is compact, then $b$ is weakly continuous — a reference request

While reading the well known book Minimax Methods in Critical Point Theory with Applications to Differential Equations by Paul Rabinowitz, in the proof of a generalisation of the Mountain Pass Theorem ...
Alexandru Pirvuceanu's user avatar
7 votes
2 answers
520 views

Weak convergence related to Hermite polynomial?

I am reading Griffiths's quantum mechanics book; in the section about harmonic oscillators, he wrote out the amplitude of wave function, and compared with the classical harmonic oscillators. He ...
Yuval's user avatar
  • 637
0 votes
0 answers
49 views

Bounding the ratio of two functions given their Laplace Transforms

Suppose $h(i)$ is a probability density that's "nice" in some sense, and $g(i)=E[f(i,x)]=\int \mathrm{d}i\ h(i)f(i,x)$ How could I bound the following ratio $r(t)$ from above? $$ r(t)=\frac{...
Yaroslav Bulatov's user avatar
2 votes
1 answer
200 views

An integral transform computation

In Erdelyi, Tables of Integral Transforms, p. 344 Section 7.2. they note that \begin{align} \frac{1}{2 \pi i} \int_{c-i\infty}^{c+i\infty} s^{\nu} e^{\alpha s^2} x^{-s} \, ds = 2^{-\nu/2} \pi^{-...
user506603's user avatar
0 votes
0 answers
42 views

dynamical system modified by PD matrix

Assume we have a dynamical system, i.e an ODE of the form $$ \frac{d\vec{x}}{dt}=g(\vec{x}) $$ which we know how to solve. Now consider a modified ODE $$ \frac{d\vec{x}}{dt}=\hat{A}g(\vec{x}) $$ with $...
Kol Namer's user avatar
18 votes
0 answers
698 views

Are these continued fractions of integrals known?

Simplified repost of Are these continued fractions of integrals known? on MSE EDIT: The period of the oscillations of $$f(s)=\dfrac1{1+\dfrac s{1+\dfrac{s^2/2!}{1+\dfrac{s^3/3!}{1+\cdots}}}}$$ ...
ə̷̶̸͇̘̜́̍͗̂̄︣͟'s user avatar
6 votes
1 answer
463 views

Cauchy-Schwarz-like inequality with a power $p$ term

We set : $\phi_1, \phi_2 : \mathbb{R} \to \mathbb{R}^M$ with compact support $\theta \in \mathbb{R}^M$ non-zero, $\theta_{p-1} = (\operatorname{sign}(\theta_i)|\theta_i|^{p-1})_{1 \leq i \leq M} \in \...
Orso Forghieri's user avatar
10 votes
7 answers
850 views

$\int_L^\infty \exp(- t - y/t) \, dt = \text{?}$

Let $y>0$, $L>0$. Has the (special?) function given by $$f(y,L) = \int_{L}^\infty e^{- t - y/t} \, dt$$ been studied? Are there precise, simple bounds? Let me try to attempt to reinvent the ...
H A Helfgott's user avatar
  • 19.4k
2 votes
1 answer
660 views

Use of Stein's maximal principle in Bourgain's paper on Besicovitch sets

I'm trying to understand Bourgain's paper "Besicovitch type maximal operators and applications to Fourier analysis". Let $\xi\in S^2\subset\mathbb{R}^3$ be a unit vector and $\delta>0$, ...
Simplyorange's user avatar
-2 votes
1 answer
109 views

convergence for a series [closed]

Show that the series $$\sum_{n=2}^{\infty}\frac{1}{[\frac{(1+\epsilon)\log n}{\log\log n}]!}$$converges for $\epsilon>0$. Stirling's approximation gives that the convergence for the series is ...
Sheng Wang's user avatar
1 vote
1 answer
190 views

Dense subset for $C_0(\mathbb{R})$

I want to know if $\left\{\frac{(1-\cos \alpha x)} {x^2}\right\}_{\alpha>0}$ is dense in $C_0(\mathbb{R})=\{f\in C(\mathbb{R})\mid \lim_{|x|\to\infty}f(x)=0\}$? That is, for any $f\in C_0(\mathbb{...
Sheng Wang's user avatar

1 2 3
4
5
70